City leader: If our city adopts the new tourism plan, the amount of money that tourists spend here annually will incr...

yrl496 on October 7 at 08:22PM

Can you explain the correct answer?

I understand why all of the answer choices are wrong but I don't fully understand why the correct answer is exactly correct. Can you please explain?

Reply
Create a free account to read and take part in forum discussions.

Already have an account? log in

Emil-Kunkin on October 9 at 01:01PM

The argument tells us that plan A would have a good impact, and that plan A would have the same good impact as plan B, which is reasonable, but A would be cheaper.

Based on this, I would want to know if there is any other reason that b is better than a. We know that the other plan is reasonable, but maybe the tourism plan is unreasonable for some reason, such as undesirable consequences.

The correct answer eliminates this possible flaw. It shows the two plants a effectively equal and eliminates the chance that the factory plan is reasonable and the tourism plan is not.